Difference between revisions of "2024 USAJMO Problems/Problem 3"

(Solution 1)
Line 6: Line 6:
 
== Solution 1 ==
 
== Solution 1 ==
  
 +
Lemma <math>1</math>:
 +
 +
Given a prime <math>p</math>, a positive integer <math>k</math>, and an even <math>m</math> such that <math>p^k|a(m)</math>, we must have that <math>p^{k+1}|a(m+2)</math>.
 +
 +
Proof of Lemma <math>1</math>:
 +
 +
<math>a(m+1)\equiv (a(m))^{m+1}-1\equiv -1 \mod p^{k+1}</math>
 +
 +
Therefore, <math>a(m+2)\equiv (a(m+1))^{m+2}-1\equiv (-1)^{m+2}-1\equiv 0 \mod p^{k+1}</math>
  
 
==See Also==
 
==See Also==
 
{{USAJMO newbox|year=2024|num-b=2|num-a=4}}
 
{{USAJMO newbox|year=2024|num-b=2|num-a=4}}
 
{{MAA Notice}}
 
{{MAA Notice}}

Revision as of 09:31, 28 March 2024

Problem

Let $a(n)$ be the sequence defined by $a(1)=2$ and $a(n+1)=(a(n))^{n+1}-1$ for each integer $n\geq1$. Suppose that $p>2$ is prime and $k$ is a positive integer. Prove that some term of the sequence $a(n)$ is divisible by $p^k$.

Solution 1

Lemma $1$:

Given a prime $p$, a positive integer $k$, and an even $m$ such that $p^k|a(m)$, we must have that $p^{k+1}|a(m+2)$.

Proof of Lemma $1$:

$a(m+1)\equiv (a(m))^{m+1}-1\equiv -1 \mod p^{k+1}$

Therefore, $a(m+2)\equiv (a(m+1))^{m+2}-1\equiv (-1)^{m+2}-1\equiv 0 \mod p^{k+1}$

See Also

2024 USAJMO (ProblemsResources)
Preceded by
Problem 2
Followed by
Problem 4
1 2 3 4 5 6
All USAJMO Problems and Solutions

The problems on this page are copyrighted by the Mathematical Association of America's American Mathematics Competitions. AMC logo.png